Sail E0 Webinar

Reasoning Aptitude

STATEMENT AND CONCLUSION MCQs

Statement Conclusions And Inferences

Total Questions : 618 | Page 32 of 62 pages
Question 311. Should rail-roads be privatised?Arguments:I. Yes, it is not possible to supervise an organization as big as a rail-road by any private company or person.II. No, privatisation makes train-journey expensive.
  1.    if only argument I is strong.
  2.    if only argument II is strong.
  3.    if either I or II is strong.
  4.    if neither I nor II is strong.
  5.    if both I and II are strong.
 Discuss Question
Answer: Option E. -> if both I and II are strong.


We cannot check the validity of the two given statements but if both are true, each will present a strong argument.


Question 312. Should the neighbouring country be given free food grain supply?Arguments:I. Yes, the country is severely affected due to a series of natural calamities.II. No, this country has always been our enemy.
  1.    if only argument I is strong.
  2.    if only argument II is strong.
  3.    if either I or II is strong.
  4.    if neither I nor II is strong.
  5.    if both I and II are strong.
 Discuss Question
Answer: Option A. -> if only argument I is strong.


"Free food grain supply" implies that the neighbouring country is in trouble. Statement I explains the troubles and is a strong argument. The fact that the other country is an enemy does not stop us from helping them as we have already understood that it is in trouble. Therefore statement II is a weak statement.


Question 313. Statement:
Every year Malaria gets rampant after the rains.Courses of Action:I. Pesticides should be properly sprayed on a regular basis.II. People should be supplied with mosquito nets and mosquito coils free by the government.III. Using various means of media, people should be informed about the methods to be adopted for prevention from these seasonal diseases.
  1.    Only I and III follow
  2.    Only II and III follow
  3.    Only II follows
  4.    Only I and II follow
  5.    All the three follow
 Discuss Question
Answer: Option A. -> Only I and III follow


Here the problem is malaria and we know that the cause of malaria is mosquito. As we know that mosquitoes can be controlled by pesticides. So, I is a proper course of action.Course of action II is not practically possible because that will cause a burden on the government. Hence it does not follow.Course of action III is feasible and also not very costly, hence it is a proper course of action. 


Question 314. Statement:Owing to rigging during polls in democratic states, the deserving candidates lose the elections.Courses of Action:I. Democracy must be abolished.II. People must be educated about the value of their vote and they should be made to cast their vote.III. Military force should be employed at every polling booth.
  1.    Only I and III follow
  2.    Only II follow
  3.    Either I or III follows
  4.    Only III follows
  5.    All the three follow
 Discuss Question
Answer: Option A. -> Only I and III follow


Here the course of action I is a negative course of action. Course of action II can reduce rigging, as it says "they should be made to cast their votes". Hence it follows.Course of action III can reduce rigging. As some authority should scrutinise the polling centers so that polling is conducted without rigging.


Question 315. Statement:
Students attendance in regular colleges is falling day by day, while in private coaching centers the situation is just the opposite.Courses of Action:I. Students must be given individual attention.II. At least once in a week they must be given a test.III. Without proper attendance, a student must not be allowed to attend the final examinations.
  1.    Only II follows
  2.    I and III follow
  3.    Only III follows
  4.    All the three follow
  5.    None follows
 Discuss Question
Answer: Option E. -> None follows


Course of action I is based on the assumption that the students are not attending the class due to lack of individual attention, hence I does not follow. For the similar reason course of action II is not proper course of action. II is not a proper course of action. Course of action III force the student to attend the class without solving the problem of not attending the classes. Hence, III does not follow.


Question 316. Statement:It is because of a multiparty system in our country that a coalition government is formed. Wherein the country suffers from instability.Courses of Action:I. There should be only a bi-party system and to this extent the constitution must be amended.II. Whenever any party is not in the majority, the elections must be held again.III. If there is no absolute majority for any national party, the president should dissolve the parliament and he should rile the country during that period.
  1.    Only II and III follow
  2.    Only I and II follow
  3.    Only I and III follow
  4.    Only I follows
  5.    None follows
 Discuss Question
Answer: Option D. -> Only I follows


Here the problem is a multiparty system which is causing instability.Course of action I is talking about bi-party system and hence this can solve the problem.Holding election for multiple number of times is not possible. Hence II does not follow.Course of action III does not solve the problem the problem may re-occur again in future.


Question 317. Statement:Devotees in the pilgrimage places are much disturbed by the loud noise made by the loud speakers and tape records played in the premises of temple streets.Courses of Action:I. The devotees should be provided with ear plugs.II. All those, who make noise by using any of the means, should be strictly warned by the government to adhere to the permissible sound levels, or else face punishment.III. Selling and buying must be prohibited in pilgrimage premises.
  1.    Only I follows
  2.    Only II follows
  3.    Only II and III follows
  4.    Only III follows
  5.    None follows
 Discuss Question
Answer: Option B. -> Only II follows


Here the problem is loud noise made by loud speakers and tape records. Hence, course of action I and II do not follow.Course of action III is a possible course of action, and this will help to reduce the loud noise to a permissible limit.


Question 318. Statement:Although we have rating agencies like Crisil, ICRA, there is demand to have a separate rating agency of IT companies to protect investors.Conclusions:I. Assessment of financial worth of IT companies calls for separate, set of skills, insight and competencies.II. Now the investors investing in IT companies will get protection of their investment.
  1.    If only conclusion I follows
  2.    If only conclusion II follows
  3.    If either I or II follows
  4.    If neither I nor II follows
  5.    If both I and II follow
 Discuss Question
Answer: Option A. -> If only conclusion I follows


II may be an assumption of the speaker. But certainly it is not a conclusion.


Question 319. Statement:During 1997-98 the total loss incurred by the 1 1 1 public sector units was to the tune of Rs. 6809 crores which has converted into paid capitals by the government of its total investment of Rs. 5129 crores.Conclusions:I. The government is left with only one option, that is, to privatise these units.II. The government did not take care in the matter of investments in these public sector units.
  1.    If only conclusion I follows
  2.    If only conclusion II follows
  3.    If either I or II follows
  4.    If neither I nor II follows
  5.    If both I and II follow
 Discuss Question
Answer: Option D. -> If neither I nor II follows


-NA-


Question 320. Statement:Population increase coupled with depleting resources is going to be the scenario of many developing countries in days to come.Conclusions:I. The population of developing countries will not continue to increase in future.II. It will be very difficult for the governments of developing countries to provide decent quality of life.
  1.    If only conclusion I follows
  2.    If only conclusion II follows
  3.    If either I or II follows
  4.    If neither I nor II follows
  5.    If both I and II follow
 Discuss Question
Answer: Option B. -> If only conclusion II follows


With the limited resources and overpopulation it is very hard to provide decent quality of life. Hence II follows.


Latest Videos

Latest Test Papers